22.07.2013 Views

Chapter 3 Elimination Theory §3. Implicitization In this section we ...

Chapter 3 Elimination Theory §3. Implicitization In this section we ...

Chapter 3 Elimination Theory §3. Implicitization In this section we ...

SHOW MORE
SHOW LESS

Create successful ePaper yourself

Turn your PDF publications into a flip-book with our unique Google optimized e-Paper software.

<strong>§3.</strong> <strong>Implicitization</strong><br />

<strong>Chapter</strong> 3 <strong>Elimination</strong> <strong>Theory</strong><br />

<strong>In</strong> <strong>this</strong> <strong>section</strong> <strong>we</strong> undertake to give a solution to the following problem:<br />

(1.0.2)<br />

The <strong>Implicitization</strong> Problem: Let Vp be a subset of k n given parametrically as<br />

x1 = f1(t1, . . . , tm),<br />

.<br />

xn = fn(t1, . . . , tm).<br />

If the fi are polynomials (or rational functions) in the variables tj, Vp will be (part of) an<br />

affine variety V . The problem is to find a system of polynomial equations in the xi that<br />

define the minimal variety V which contains Vp, i.e. for which Vp ⊂ V .<br />

Polynomial Parametrizations<br />

Example 2.8.0.5. Consider the tangent surface of the twisted cubic in R3 . The twisted cubic in R3 has parametric form R(t) = (t, t2 , t3 ). Its tangent at t is •<br />

R(t) = (1, 2t, 3t3 ); so the tangent surface of the<br />

twisted cubic has parametrization (t, u) ↦→ R(t) + u •<br />

R(t). Componentwise <strong>this</strong> surface is parametrized by<br />

x = t + u,<br />

y = t 2 + 2tu,<br />

z = t 3 + 3t 2 u.<br />

To eliminate t, u compute a Gröbner basis G for the associated ideal relative to lex order with the variables<br />

ordered by t > u > x > y > z. Using Mathematica 3.0 the result is<br />

G = {−3 x 2 y 2 + 4 y 3 + 4 x 3 z − 6 x y z + z 2 ,<br />

2 u y 3 + x y 3 − 4 x 2 y z + 5 y 2 z − 2 u z 2 − 2 x z 2 ,<br />

− 2 u y 2 − x y 2 + 2 u x z + 2 x 2 z − y z,<br />

u x y − x 2 y + 2 y 2 − u z − x z,<br />

2 u x 2 − 2 x 3 − 2 u y + 3 x y − z,<br />

u 2 − x 2 + y, t + u − x}.<br />

Only one of these Gröbner basis elements, the first one listed which <strong>we</strong> refer to as g1, doesn’t involve t or u.<br />

Setting it equal to zero gives<br />

(2.8.0.6) −3 x 2 y 2 + 4 y 3 + 4 x 3 z − 6 x y z + z 2 = 0<br />

which is the equation of a surface containing the tangent surface to the twisted cubic. Of course as far as <strong>we</strong><br />

know at present there may be points on (2.8.0.6) which aren’t on the tangent surface to the twisted cubic,<br />

and <strong>we</strong> do not know that (2.8.0.6) describes the smallest variety which contains the twisted cubic.<br />

The Basic Maps. Starting with the system (1.0.2) above, let t = (t1, . . . , tm) and<br />

f(t) = (f1(t), . . . , fn(t)). let i: k m → k n+m be the map t ↦→ (t, f(t)) and let πm : k n+m → k n be the map<br />

(a1, . . . , an+m) ↦→ (am+1, . . . , an+m). F : k m → k n is the map F = πm ◦ i or t ↦→ f(t). The coordinate<br />

functions on k m+n will be denoted, respectively, by t1, . . . , tm, x1, . . . , xn. At the risk of some ambiguity <strong>we</strong><br />

will denote the function (t1, . . . , tm, x1, . . . , xn) ↦→ fi(t1, . . . , tm) by fi also, leaving it to the reader to infer<br />

the interpretation from the context. <strong>In</strong> a slight extension of the preceding notation x = (x1, . . . , , xn) and<br />

x − f(t) will refer either (i) to the apparent n-vector valued function on k m+n or without ambiguity to (ii)<br />

the set of functions {x1 − f1(t1, . . . , tm), . . . , xn − fn(t1, . . . , tm)} on k m+n . When the fi’s are polynomials<br />

I = 〈x − f(t)〉 will thus denote the ideal in k[t1, . . . , tm, x1, . . . , xn] = k[t, x] generated by the polynomials<br />

xi − fi(t), 1 ≤ i ≤ n.<br />

1


Elementary Observation. i(k m ) = V(I).<br />

Proof. (⊂): Suppose t ∈ k m , then i(t) = (t, f(t)). Call <strong>this</strong> point q ∈ k m+n . Then x(q) = f(t) and<br />

f(q) = f(t) also (when the fi’s are regarded as functions on k m+n . Thus x(q) − f(q) = 0 and q ∈ V(I).<br />

(⊃): If (t, y) ∈ V(I), then for each 1 ≤ i ≤ n, 0 = (xi − fi)(t, y) = yi − fi(t); so y = f(t) and our point<br />

has the form (t, f(t)) which puts it in i(k m ).<br />

One consequence of <strong>this</strong> observation is that i(k m ) is an affine variety. We will denote it by V = i(k m ).<br />

Our goal is to describe the smallest affine variety in k n which contains πm(V ) = F (k m ).<br />

Theorem 3.3.1 (Polynomial <strong>Implicitization</strong>). If k is an infinite field, let F : k m → k n be defined<br />

as above where <strong>we</strong> suppose that fi ∈ k[t1, . . . , tm]. Let I be the ideal I = 〈x1 − f1, . . . , xn − fn〉 ⊂<br />

k[t1, . . . , tm, x1, . . . , xn] and let Im = I ∩ k[x1, . . . , xn] be the m-th elimination ideal. Then V(Im) is the<br />

smallest variety in k n containing F (k m ).<br />

Proof. Since V(g1, . . . , gr) = V(g1) ∩ · · · ∩ V(gr) when g1, . . . , gr ∈ k[x1, . . . , xn], our conclusion is<br />

equivalent to the assertion: “If a polynomial g ∈ k[x1, . . . , xn] vanishes on F (k m ), then it vanishes on<br />

V(Im).” This is just statement (i) of the Closure Theorem when k is algebraically closed and <strong>we</strong> have<br />

deferred its proof to later. Taking it as known when k is algebraically closed, <strong>we</strong> must prove it in general<br />

whenever k is an infinite field.<br />

To do <strong>this</strong> <strong>we</strong> need some preparation. Let K be an algebraically closed extension field of k. Start with<br />

{x − f(t)} and using these polynomials, whose coefficients are in k, and a monomial ordering in which the<br />

ti’s come before the xj’s, compute a Gröbner basis G for I. G is unaffected by whether the computations are<br />

carried out using k as the field or using K because computing a Gröbner basis will never take us out of the field<br />

of coefficients of the starting ideal basis. We will use Ĩ and Ĩm to indicate the ideal in K[t1, . . . , tm, x1, . . . , xn]<br />

generated by the polynomials {x−f(t)} and the corresponding m-th elimination ideal of Ĩ, respectively. Then<br />

a basis for Im consists of G∩k[x1, . . . , xn] and a basis for Ĩm consists of G∩K[x1, . . . , xn]. Which is, of course,<br />

the same as G ∩ k[x1, . . . , xn]. Nowhere does it make any difference whether the computations are carried<br />

out over k or over K. This will produce polynomials b1(x), . . . , bs(x) in k[x1, . . . , xn] with the property that<br />

( Ĩm) Ĩm = { ˜ ψ ∈ K[x]: ˜ ψ(x) = ˜ϕ1(t, x)b1(x) + · · · + ˜ϕs(t, x)bs(x), ˜ϕi(t, x) ∈ K[t, x], 1 ≤ i ≤ s}<br />

and<br />

(Im) Im = {ψ ∈ k[x]: ψ(x) = ϕ1(t, x)b1(x) + · · · + ϕs(t, x)bs(x), ϕi(t, x) ∈ k[t, x], 1 ≤ i ≤ s}.<br />

The only restriction on the ˜ϕi(t, x)’s and the ϕj(t, x)’s is that the variables t1, . . . , tm cancel out in the<br />

expressions for ˜ ψ and ψ above. This is easily seen to be equivalent to saying<br />

( Ĩm) Ĩm = { ˜ ψ ∈ K[x]: ˜ ψ(x) = ˜ h1(x)b1(x) + · · · + ˜ hs(x)bs(x), ˜ hi(x) ∈ K[x], 1 ≤ i ≤ s}<br />

and<br />

(Im) Im = {ψ ∈ k[x]: ψ(x) = h1(x)b1(x) + · · · + hs(x)bs(x), hi(x) ∈ k[x], 1 ≤ i ≤ s}.<br />

Using the fact that these hi’s and ˜ hj’s can be chosen rather arbitrarily, <strong>we</strong> have established that<br />

and<br />

a point q ∈ K n is in V( Ĩm) iff bi(q) = 0, 1 ≤ i ≤ s<br />

a point p ∈ k n is in V(Im) iff bi(p) = 0, 1 ≤ i ≤ s.<br />

2


Now to the proof proper. When g ∈ k[x1, . . . , xn] each line below implies the line which follows it.<br />

g vanishes on F (k m );<br />

g ◦ F vanishes on k m ;<br />

g ◦ F is the zero polynomial ;<br />

g ◦ F vanishes on K m ;<br />

g vanishes on F (K m ); <strong>this</strong> implies the next line by Statement (i) of the Closure Theorem;<br />

g vanishes on {q ∈ K n : Ĩm(q) = {0}};<br />

g vanishes on {q ∈ K n : b1(q) = 0, . . . , bs(q) = 0};<br />

g vanishes on {p ∈ k n : b1(p) = 0, . . . , bs(p) = 0};<br />

g vanishes on {p ∈ k n : Im(p) = {0}};<br />

V(Im) ⊂ V(g). where the varieties are in k n .<br />

The implicitization problem is then solved by the following recipe: Form the ideal<br />

I = 〈xi − fi(t1, . . . , tm): 1 ≤ i ≤ n〉 and choose a Gröbner basis G with respect to a monomial order in<br />

which the t’s precede the x’s.. The polynomials in G which don’t involve t1, . . . , tm, say Gm, then form a<br />

basis for the ideal Im, and the minimal variety containing the range F (k m ) = {f(t): t ∈ k m } is the variety<br />

V(Im) = V(Gm).<br />

This was precisely the procedure follo<strong>we</strong>d to get (2.8.0.6), and it shows that the minimal variety in R 3<br />

containing the tangent surface to the twisted cubic is the locus of points (x, y, z) satisfying g1(x, y, z) = 0 or<br />

(2.8.0.6) −3 x 2 y 2 + 4 y 3 + 4 x 3 z − 6 x y z + z 2 = 0.<br />

We still don’t know, ho<strong>we</strong>ver, if the twisted cubic actually fills up <strong>this</strong> locus. To ans<strong>we</strong>r <strong>this</strong> question <strong>we</strong> must<br />

see whether each partial solution (x, y, z) ∈ V(g), where g is the polynomial on the lefthand side of (2.8.0.6),<br />

extends to (t, u, x, y, z) ∈ V(I) which is the graph of the tangent space to the twisted cubic. We work first<br />

over C, so that <strong>we</strong> can use the Extension Theorem. As usual <strong>we</strong> will extend by adding one component at a<br />

time.<br />

Suppose (x, y, z) ∈ V(I2) which is V(g1). If the variables are ordered t > u > x > y > z, I2 = I ∩k[x, y, z]<br />

is the first elimination ideal of I1 = I ∩ k[u, x, y, z]. We also know from the <strong>Elimination</strong> Theorem that I1 is<br />

the ideal in C[u, x, y, z] generated by the basis<br />

G = {−3 x 2 y 2 + 4 y 3 + 4 x 3 z − 6 x y z + z 2 ,<br />

2 u y 3 + x y 3 − 4 x 2 y z + 5 y 2 z − 2 u z 2 − 2 x z 2 ,<br />

− 2 u y 2 − x y 2 + 2 u x z + 2 x 2 z − y z,<br />

u x y − x 2 y + 2 y 2 − u z − x z,<br />

2 u x 2 − 2 x 3 − 2 u y + 3 x y − z,<br />

u 2 − x 2 + y}.<br />

The coefficients of the leading terms in u are<br />

− 3 x 2 y 2 + 4 y 3 + 4 x 3 z − 6 x y z + z 2 ,<br />

2y 3 − 2z 2 ,<br />

2xz − 2y 2 ,<br />

xy − z,<br />

2x 2 − 2y,<br />

1.<br />

These are the gi’s in the Extension Theorem. Since the variety they generate is ∅, every partial solution in<br />

I2 extends to a partial solution (u, x, y, z) ∈ V(I1). Further, if (x, y, z) ∈ R 3 , then (u, x, y, z) ∈ R 4 as is easily<br />

seen by noticing that the second polynomial in G above only involves u to the first po<strong>we</strong>r.<br />

3


Let’s see if <strong>we</strong> can extend another step from V(I1) to V(I). Here<br />

G = {−3 x 2 y 2 + 4 y 3 + 4 x 3 z − 6 x y z + z 2 ,<br />

2 u y 3 + x y 3 − 4 x 2 y z + 5 y 2 z − 2 u z 2 − 2 x z 2 ,<br />

− 2 u y 2 − x y 2 + 2 u x z + 2 x 2 z − y z,<br />

u x y − x 2 y + 2 y 2 − u z − x z,<br />

2 u x 2 − 2 x 3 − 2 u y + 3 x y − z,<br />

u 2 − x 2 + y,<br />

t + u − x}.<br />

forms a basis and the last of these basis elements has a coefficient of the highest po<strong>we</strong>r of t which is 1 and<br />

thus never vanishes. Again the Extension Theorem (or Corollary 3.0.4) guarantees that any partial solution<br />

(u, x, y, z) ∈ V(I1) extends to (t, u, x, y, z) ∈ V(I). Furthermore, (t, u, x, y, z) ∈ R 5 if (u, x, y, z) ∈ R 4 , seen<br />

here by looking at the polynomial t + u − x. Thus F (k 5 ) = V(I2) and the parametrization fills up the variety.<br />

Rational Parametrizations<br />

Example 3.3.2 Consider the following rational parametrization:<br />

(3.3.3) (u, v) F<br />

<br />

2 u v2<br />

↦→ , , u .<br />

v u<br />

Treating <strong>this</strong> as in the polynomial case <strong>we</strong> would introduce the coordinate variables (x, y, z) and write <strong>this</strong><br />

in the form<br />

Then <strong>we</strong> would consider the two maps<br />

and<br />

i<br />

x = u2<br />

v ,<br />

y = v2<br />

u ,<br />

z = u.<br />

k 2<br />

−→ k 5<br />

<br />

(u, v) ↦→ u, v, u2<br />

<br />

v2<br />

, , u<br />

v u<br />

k 5<br />

π2<br />

−→ k 3<br />

(u, v, x, y, z) ↦→ (x, y, z),<br />

whose composition is the map F of (3.3.3). The variety V ⊂ k5 should be something like<br />

V ′<br />

<br />

x − u2<br />

<br />

v2<br />

v , y − u , z − u , but since the relations defining V are not then polynomial relations <strong>we</strong> would<br />

modify them by clearing denominators to V vx − u2 , uy − v2 , z − u . If <strong>we</strong> set I = 〈vx−u 2 , uy −v 2 , z −u〉,<br />

a Gröbner basis for I computed relative to lex order with u > v > x > y > z is<br />

G = {x 2 y z − z 4 , −x y z + v z 2 , v x − z 2 , v 2 − y z, u − z}.<br />

To eliminate u, v <strong>we</strong> look at the second elimination ideal I2 = I ∩ k[x, y, z]. According to our results a<br />

Gröbner basis for I2 is given by G ∩ k[x, y, z] which in <strong>this</strong> case yields I2 = 〈x2 y z − z4 <br />

〉. The associated<br />

2<br />

u<br />

variety is V(I2) = V(x2y − z3 v2<br />

) ∪ V(z). Now the set v , u , u : uv = 0 is actually a subset of V(x2y − z3 )<br />

alone, and there are in fact no points in it which are in V(z); so to get the minimal variety containing the<br />

image set of F <strong>we</strong> certainly don’t want to include the V(z). What <strong>this</strong> says is that <strong>we</strong> have to be a bit more<br />

sophisticated.<br />

4


and<br />

We do <strong>this</strong> by including an extra variable as follows: Consider the two maps<br />

j<br />

k 2<br />

−→ k 6<br />

<br />

(u, v) ↦→ uv, u, v, u2<br />

<br />

v2<br />

, , u<br />

v u<br />

k 6<br />

π3<br />

−→ k 3<br />

(t, u, v, x, y, z) ↦→ (x, y, z),<br />

J is now defined as the ideal J = 〈t−uv, vx−u 2 , uy −v 2 , z −u〉 (REMARK!!! It makes no difference in H<br />

and seems to make better sense for what follows if J is defined by J = 〈1−tuv, vx−u 2 , uy −v 2 , z −u〉). For<br />

either choice of J the Gröbner basis H for <strong>this</strong> ideal computed relative to lex order with t > u > v > x > y > z<br />

is<br />

H = {x 2 y − z 3 , −x y + v z, v x − z 2 , v 2 − y z, u − z, −x + t z 3 , −v + t y z 2 , −1 + t x y}.<br />

The third elimination ideal J3 = J ∩ k[x, y, z] now has Gröbner basis {x2 y − z3 } and the variety V(x2y − z3 <br />

)<br />

2<br />

u v2<br />

is in fact the minimal variety containing the set of points v , u , u : uv = 0 . We give a slightly varied<br />

description: Let F = π3 ◦ j and let W = {(u, v): uv = 0} be the set where at least one of the denominators<br />

is zero. Then V(J3) is the smallest variety in k3 which contains the set F (k2 − W ). This is the content of<br />

the following theorem.<br />

Theorem 3.3.4 (Theorem 2) (Rational <strong>Implicitization</strong>). If k is an infinite field, consider the<br />

rational parametrization F : (k m − W ) → k n given by<br />

(t1, . . . , tm) F<br />

<br />

f1(t1, . . . , tm)<br />

↦→<br />

g1(t1, . . . , tm) , . . . , fn(t1,<br />

<br />

. . . , tm)<br />

,<br />

gn(t1, . . . , tm)<br />

where fi, gi ∈ k[t1, . . . , tm], and W = V(g1(t) · · · gn(t)). Let J be the ideal<br />

J = 〈g1(t)x1 − f1(t), . . . , gn(t)xn − fn(t), 1 − yg1(t) · · · gn(t)〉 ⊂ k[y, t1, . . . , tm, x1, . . . , xn],<br />

and let Jm+1 = J ∩ k[x1, . . . , xn] be the (m + 1)-st elimination ideal. Then V(Jm+1) is the smallest variety<br />

in k n containing F (k m − W ).<br />

Proof. Either taken as similar to the proof of the Polynomial <strong>Implicitization</strong> Theorem or deferred.<br />

<strong>Chapter</strong> 3 §3 Exercises<br />

<strong>§3.</strong>3.1.<br />

Prove the “Elementary Observation” that i(k m ) = V(I).<br />

Solution. This is done in the text <strong>section</strong> <strong>§3.</strong>3.0.<br />

<strong>§3.</strong>3.2.<br />

When k = C, the conclusion of Theorem 1 can be strengthened. Namely, one can show that there is a<br />

variety W ⊂<br />

= V(Im) such that V(Im) − W ⊂ F (Cm ). Prove <strong>this</strong> using the Closure Theorem.<br />

Solution. The Closure Theorem guarantees the existence of an affine variety W ⊂<br />

= V(Im) such V(Im)−W ⊂<br />

πm(V ). We know that πm(V ) = F (C m ). Putting <strong>this</strong> together with the statement in the previous sentence<br />

completes the argument.<br />

5


<strong>§3.</strong>3.3.<br />

Give an example to show that Exercise 3.3.2 is false over R. Hint: t 2 is always positive.<br />

Solution. Consider the parametrization t ↦→ t 2 mapping R 1 → R 1 . The map i: R → R 2 is the map<br />

t ↦→ (t, t 2 ) and its image i(R 1 ) is the variety V (V is just the parabola with equation x = t 2 if t, x denote<br />

the coordinate functions in R 2 .) Now F (R) = {x ∈ R: x ≥ 0}. I = 〈x − t 2 〉 and the only polynomial in <strong>this</strong><br />

ideal which does not involve t is 0; so I1 = {0} and V(I1) = R.<br />

If Exercise 3.3.2 <strong>we</strong>re true for <strong>this</strong> example there would be a variety W ⊂<br />

= R such that R−W ⊂ {x ∈ R: x ≥<br />

0}. This means that W ⊃ {x ∈ R: x < 0}. The polynomials which vanish on W would have to vanish on<br />

the negative reals. Only the zero polynomial does <strong>this</strong>; so W must contain R contradicting the assumption<br />

that W ⊂<br />

= R. No such W can exist.<br />

<strong>§3.</strong>3.4.<br />

<strong>In</strong> the text <strong>we</strong> proved that over C the tangent surface to the twisted cubic is defined by the equation<br />

(2.8.0.6) −3 x 2 y 2 + 4 y 3 + 4 x 3 z − 6 x y z + z 2 = 0.<br />

We want to show that the same is true over R. If (x, y, z) is a real solution of the above equation, then <strong>we</strong><br />

proved (using the Extension Theorem) that there are t, u ∈ C such that<br />

(3.3.4.1)<br />

x = t + u,<br />

y = t 2 + 2tu,<br />

z = t 3 + 3t 2 u.<br />

Use the Gröbner basis given in the text to show that t and u are real. This will prove that (x, y, z) is on the<br />

tangent surface in R 3 . Hint: First show that u is real.<br />

Solution. The ideal I = 〈x − t − u, y − t 2 − 2tu, z − t 3 − 3t 2 u〉 has Gröbner basis<br />

G = {−3 x 2 y 2 + 4 y 3 + 4 x 3 z − 6 x y z + z 2 ,<br />

2 u y 3 + x y 3 − 4 x 2 y z + 5 y 2 z − 2 u z 2 − 2 x z 2 ,<br />

− 2 u y 2 − x y 2 + 2 u x z + 2 x 2 z − y z,<br />

u x y − x 2 y + 2 y 2 − u z − x z,<br />

2 u x 2 − 2 x 3 − 2 u y + 3 x y − z,<br />

u 2 − x 2 + y, t + u − x}.<br />

If (x, y, z) ∈ R 3 the vanishing of the second polynomial in G shows that u is real and then the vanishing of<br />

the last polynomial in G shows that t is real.<br />

<strong>In</strong> summary. Every (x, y, z) which is complex and satisfies (2.8.0.6) can be augmented to (t, u, x, y, z),<br />

a point in V and hence a zero of the polynomials in G, by adjoining a pair of complex numbers, t, u. Now<br />

if (x, y, z) are real to start with then t, u are necessarily real also and it follows that a real triple (x, y, z)<br />

satisfying (2.8.0.6) has the form (3.3.4.1) for suitable choice of (real) t, u. What <strong>we</strong> have shown is that the<br />

locus (2.8.0.6) and (3.3.4.1) describe exactly the same point set, and <strong>this</strong> happens whether the coordinates<br />

have values in C or in R.<br />

<strong>§3.</strong>3.5.<br />

<strong>In</strong> the parametrization of the tangent surface of the twisted cubic, show that the parameters t and u<br />

are uniquely determined by x, y, and z. Hint: The argument is similar to what you did in Exercise 4.<br />

Solution. Suppose (t, u, x, y, z) satisfy (3.3.4.1). Then they constitute a zero of the polynomials in G. u is<br />

determined by (x, y, z) from the second equation and t is then determined by u, x from the last equation.<br />

6


<strong>§3.</strong>3.6.<br />

Let S be the parametric surface defined by<br />

(3.3.6.1)<br />

x = uv,<br />

y = u 2 ,<br />

z = v 2 .<br />

(a) Find the equation of the smallest variety V that contains S.<br />

Solution. By Theorem 3.0.1, the Polynomial <strong>Implicitization</strong> Theorem, <strong>this</strong> is the variety V(I2), where<br />

I = 〈x − uv, y − u 2 , z − v 2 〉 and the variables are ordered u > v > x > y > z.<br />

Using Mathematica:<br />

H={x-u*v, y-u^2, z-v^2}<br />

follo<strong>we</strong>d by<br />

GroebnerBasis[H,{u,v,x,y,z}, MonomialOrder->{{1,1,1,1,1},<br />

{0,0,0,0,-1},{0,0,0,-1,0},{0,0,-1,0,0},{0,-1,0,0,0}}]<br />

which is the DegreeReverseLexicographic order or just using the Lexicographic order in the command<br />

GroebnerBasis[H,{u,v,x,y,z}]<br />

Leads to the output<br />

(3.3.6.2) {x 2 − y z, v x − u z, u x − v y, v 2 − z, u v − x, u 2 − y}<br />

and from <strong>this</strong> it follows that V has equation x 2 − yz = 0.<br />

(b) Over C, use the Extension Theorem to prove that S = V . Hint: The argument is similar to what<br />

<strong>we</strong> did for the tangent surface of the the twisted cubic.<br />

Solution. What <strong>we</strong> have to do is to show that every (x, y, z) which satisfies x 2 − yz = 0 can be written in<br />

the form (3.3.6.1) for some choice of u, v. Since <strong>we</strong> know (3.3.6.1) and the vanishing of the expressions in<br />

(3.3.6.2) say exactly the same things about (u, v, x, y, z) <strong>we</strong> will work with (3.3.6.2).<br />

A Direct Argument. Given (x, y, z) with x 2 − yz = 0, choose u ∈ {± √ y} and v ∈ {± √ z}. Then<br />

(uv) 2 = yz = x 2 ; so uv = ±x. Choose the signs so that uv = x. It follows that uv = x. All the<br />

expressions of (3.3.6.2) are thus zero except possibly for the second and third. Now vx = v · uv = uv 2 = uz<br />

and ux = u 2 v = yv; so the second and third expressions are zero too. Thus <strong>this</strong> choice of u, v leads to<br />

(u, v, x, y, z) satisfying (3.3.6.2) and hence (3.3.6.1).<br />

Using the Extension Theorem. If <strong>we</strong> eliminate u first to get to I1 and then v to get to I2 <strong>we</strong> have<br />

the Gröbner bases<br />

(3.3.6.3)<br />

I = 〈x 2 − yz, u 2 − y, −vx + uz, ux − vy, uv − x, v 2 − z〉,<br />

I1 = 〈x 2 − yz, v 2 − z〉,<br />

I2 = 〈x 2 − yz〉.<br />

Now the Extension Theorem guarantees that given (x, y, z) ∈ V(I2), or equivalently satisfying x 2 − yz = 0,<br />

there is a v such that (v, x, y, z) ∈ V(I1) because the set of leading coefficients of v in the polynomials<br />

of the given basis for I1 is {x 2 − yz, 1} and they have no common (x, y, z)-zero. Similarly, making another<br />

application of the Extension Theorem, <strong>this</strong> time from V(I1) to V(I), <strong>we</strong> see that the set of leading coefficients<br />

of u in the polynomials in the given basis for I contains a 1, so that they don’t have a common zero. This<br />

means that <strong>we</strong> can find a u such that (u, v, x, y, z) ∈ V(I). This (u, v, x, y, z) satisfies (3.3.6.1) and shows<br />

that S = V .<br />

(c) Over R, show that S only covers the “half” of V . What parametrization would cover the other half?<br />

Solution. Looking at the “Direct Argument” in the solution to part (b) of <strong>this</strong> Exercise 3.3.6, <strong>we</strong> see that<br />

<strong>we</strong> can find real u and v only when y and z are both nonnegative. The equation x 2 = yz shows that y and<br />

z both have the same sign which leaves the “half” of V where y and z are both negative that is not covered<br />

by the parametrization (3.3.6.1).<br />

7


Consider then the parametrization<br />

(3.3.6.4)<br />

x = uv,<br />

y = −u 2 ,<br />

z = −v 2 .<br />

A Gröbner basis for J = 〈x − uv, y + u 2 , z + v 2 〉 is given by<br />

K = {x 2 − y z, v 2 + z, v x + u z, u x + v y, u v − x, u 2 + y}.<br />

So the smallest variety which contains the points (x, y, z) having the form (3.3.6.4) is still V , the locus of<br />

x 2 − yz = 0. This time <strong>we</strong> choose u ∈ {± √ −y} and v ∈ {± √ −z} with signs chosen so that uv = x as<br />

before. We get an (u, v, x, y, z) for which the expressions of K are zero and which, accordingly, satisfies<br />

(3.3.6.4). Thus (3.3.6.4) is the parametrization which fills out the other “half” of V .<br />

Remark. Make the coordinate change s = z+y<br />

√ , t =<br />

2 z−y<br />

√ , or y =<br />

t s−t √ , z =<br />

2 s+t √ . The s, t, x coordinate<br />

2<br />

axes are obtained by rotating the y, z, x axes π<br />

4 radians in the “counterclockwise” (when vie<strong>we</strong>d from the<br />

positive side of the x axis) sense about the x-axis. The variety x2−yz = 0 has s, t, x-equation x2 <br />

2 2<br />

s −t − 2 = 0<br />

or 2x2 + t2 = s2 . This is an “eliptical cone” whose central axis is the s-axis, i.e. the line (t = 0, x = 0) or,<br />

equivalently, (y = z, x = 0). Its inter<strong>section</strong> with the yz-coordinate plane is the union of the lines y = 0,<br />

x = 0 and z = 0, x = 0. The portions where (s ≥ 0 or z ≥ 0, y ≥ 0) and (s ≤ 0 or z ≤ 0, y ≤ 0) are<br />

the two “nappes” of <strong>this</strong> cone. <strong>In</strong> the complex case (3.3.6.1) covers both these nappes. <strong>In</strong> the real case<br />

they are parametrized separately with (3.3.6.1) parametrizing the portion where y ≥ 0, z ≥ 0 and (3.3.6.4)<br />

parametrizing the nappe where y ≤ 0, z ≤ 0.<br />

<strong>§3.</strong>3.7.<br />

Let S be the parametric surface<br />

(3.3.7.1)<br />

x = uv,<br />

y = uv 2 ,<br />

z = u 2 .<br />

(a) Find the equation of the smallest variety V that contains S.<br />

Solution. We know that if I = 〈x − uv, y − uv 2 , z = u 2 〉 and the variables are ordered u > v > x > y > z,<br />

then V = V(I2). Mathematica shows a Gröbner basis for I to be<br />

G = {x 4 − y 2 z, −x 3 + v y z, v x − y, −x 2 + v 2 z, −x 2 + u y, u x − v z, u v − x, u 2 − z}.<br />

We read directly from G that V = V(x 4 − y 2 z). So the equation of V is x 4 − y 2 z = 0 . From <strong>this</strong> <strong>we</strong> can<br />

easily extract the elimination ideals<br />

I = 〈x 4 − y 2 z, −x 3 + v y z, v x − y, −x 2 + v 2 z, −x 2 + u y, u x − v z, u v − x, u 2 − z〉<br />

I1 = 〈x 4 − y 2 z, −x 3 + v y z, v x − y, −x 2 + v 2 z〉<br />

I2 = 〈x 4 − y 2 z〉<br />

(b) Over C, show that V contains points which are not on S. Determine exactly which points of V are not<br />

on S. Hint: Use lexicographic order with u > v > x > y > z.<br />

Solution. What <strong>we</strong> are asked to do is find those points (x, y, z) with x 4 − y 2 z = 0 for which there are no<br />

auxiliary variables u, v such that (u, v, x, y, z) is a zero of each of the polynomials in G.<br />

8


If xy = 0 then the point<br />

in order,<br />

x 2<br />

y<br />

<br />

y<br />

, x , x, y, z ∈ V(I) as <strong>we</strong> now verify. Taking the equations after the first<br />

−x 3 + vyz = −x 3 + y 1<br />

· yz =<br />

x x · x 4 − y 2 z = 0,<br />

vx − y = y<br />

· x − y = 0,<br />

x<br />

−x 2 + v 2 z = −x 2 + y2 1<br />

z =<br />

x2 x2 (−x4 + yz) = 0,<br />

−x 2 + uy = −x 2 + x2<br />

y<br />

ux − vz = x2 y<br />

· x −<br />

y x<br />

u 2 2 2 x<br />

− z =<br />

y<br />

· y = 0,<br />

· z = 1<br />

x 2 y (x4 − y 2 z) = 0,<br />

− z = 1<br />

y 2 · (x4 − y 2 z) = 0.<br />

This leaves just the points on the xz and yz coordinate planes for consideration. If x = 0, the relation<br />

x 4 − y 2 z = 0 becomes y 2 z = 0 and <strong>we</strong> need only consider points on the y or z axes. The relations for u, v<br />

become<br />

(3.3.7.2a)<br />

(3.3.7.2b)<br />

(3.3.7.2c)<br />

(3.3.7.2d)<br />

y 2 z = 0,<br />

vyz = 0,<br />

y = 0,<br />

v 2 z = 0,<br />

uy = 0,<br />

vz = 0,<br />

uv = 0,<br />

u 2 − z = 0.<br />

Case I (y = 0): If it turns out that y = 0, <strong>we</strong> can satisfy <strong>this</strong> system by choosing v = 0 and u = √ z.<br />

That is, the point ( √ z, 0, 0, 0, z) ∈ V(I) and projects (under π2) onto (x, y, z) = (0, 0, z).<br />

Case II (y = 0, z = 0): Here there is trouble. (0, y, 0) ∈ V(I2) and yet there is no point (u, v, 0, y, 0)<br />

which is in V(I). So the portion of V which is not on S consists of the non zero points on the y-axis.<br />

Remark. The surface x 4 − y 2 z = 0 is the union of the two surfaces x 2 − y √ z = 0 and x 2 + y √ z = 0.<br />

Slicing it with the plane z = const will give two butting parabolas when const > 0, the y-axis when const = 0<br />

and the empty set when const < 0. The parabolas flatten out and approach the line y = 0 as const ↑ +∞.<br />

The trace on the plane x = x0 is the curve z = which splits into two hyperbola-like components.<br />

(x0) 4<br />

y 2<br />

<strong>§3.</strong>3.8.<br />

The Enneper surface is defined parametrically by<br />

(3.3.8.1)<br />

x = 3u + 3uv 2 − u 3 ,<br />

y = 3v + 3u 2 v − v 3 ,<br />

z = 3u 2 − 3v 2 .<br />

(a) Find the equation of the smallest variety V containing the Enneper surface. It will be a very<br />

complicated equation!<br />

9


Solution. Letting I = 〈x − 3u − 3uv 2 − u 3 , y − 3v + 3u 2 v − v 3 , z − 3u 2 − 3v 2 〉 and calculating a Gröbner<br />

basis for I using lex order with u > v > x > y > z gives<br />

(g1)<br />

(g2)<br />

(g3)<br />

(g4)<br />

(g5)<br />

(g6)<br />

G = {57395628 x 2 + 2125764 x 4 + 19683 x 6 + 57395628 y 2 − 12754584 x 2 y 2 +<br />

59049 x 4 y 2 + 2125764 y 4 + 59049 x 2 y 4 + 19683 y 6 − 172186884 z+<br />

57395628 x 2 z + 1121931 x 4 z + 57395628 y 2 z − 6259194 x 2 y 2 z+<br />

1121931 y 4 z − 229582512 z 2 + 24091992 x 2 z 2 + 196830 x 4 z 2 +<br />

24091992 y 2 z 2 − 1023516 x 2 y 2 z 2 + 196830 y 4 z 2 − 131797368 z 3 +<br />

5432508 x 2 z 3 + 10935 x 4 z 3 + 5432508 y 2 z 3 − 56862 x 2 y 2 z 3 +<br />

10935 y 4 z 3 − 42515280 z 4 + 691092 x 2 z 4 + 691092 y 2 z 4 −<br />

8424324 z 5 + 46656 x 2 z 5 + 46656 y 2 z 5 − 1049760 z 6 +<br />

1296 x 2 z 6 + 1296 y 2 z 6 − 80352 z 7 − 3456 z 8 − 64 z 9 ,<br />

− 354294 x 2 − 6561 x 4 − 8503056 v y+<br />

2480058 y 2 − 39366 x 2 y 2 + 314928 v y 3 − 32805 y 4 + 1062882 z−<br />

216513 x 2 z − 729 x 4 z − 6613488 v y z + 1436859 y 2 z − 5832 x 2 y 2 z+<br />

104976 v y 3 z − 5103 y 4 z + 944784 z 2 − 50301 x 2 z 2 − 2020788 v y z 2 +<br />

303993 y 2 z 2 + 8748 v y 3 z 2 + 341172 z 3 − 5103 x 2 z 3 −<br />

303264 v y z 3 + 27945 y 2 z 3 + 64152 z 4 − 189 x 2 z 4 − 22356 v y z 4 +<br />

945 y 2 z 4 + 6642 z 5 − 648 v y z 5 + 360 z 6 + 8 z 7 ,<br />

13122 v + 243 v x 2 − 4374 y + 81 x 2 y−<br />

729 v y 2 + 81 y 3 + 8748 v z + 27 v x 2 z − 1944 y z − 135 v y 2 z+<br />

2106 v z 2 − 270 y z 2 + 216 v z 3 − 12 y z 3 + 8 v z 4 ,<br />

39366 x 2 + 729 x 4 + 1180980 v y+<br />

4374 v x 2 y − 354294 y 2 + 5832 x 2 y 2 − 48114 v y 3 + 5103 y 4 − 118098 z+<br />

19683 x 2 z + 761076 v y z − 155277 y 2 z − 8748 v y 3 z − 91854 z 2 + 3402 x 2 z 2 +<br />

177876 v y z 2 − 21384 y 2 z 2 − 27702 z 3 + 189 x 2 z 3 + 17820 v y z 3 −<br />

945 y 2 z 3 − 4050 z 4 + 648 v y z 4 − 288 z 5 − 8 z 6 ,<br />

− 6377292 v + 2187 v x 4 + 2125764 y − 157464 x 2 y − 729 x 4 y − 1180980 v y 2 +<br />

393660 y 3 − 5832 x 2 y 3 + 50301 v y 4 − 5103 y 5 − 7085880 v z + 2125764 y z−<br />

65610 x 2 y z − 708588 v y 2 z + 161838 y 3 z + 8748 v y 4 z−<br />

3228012 v z 2 + 826686 y z 2 − 8019 x 2 y z 2 − 157464 v y 2 z 2 +<br />

21141 y 3 z 2 − 769824 v z 3 + 157464 y z 3 − 189 x 2 y z 3 −<br />

15876 v y 2 z 3 + 945 y 3 z 3 − 101088 v z 4 + 15066 y z 4 −<br />

648 v y 2 z 4 − 6912 v z 5 + 648 y z 5 − 192 v z 6 + 8 y z 6 ,<br />

972 v 2 + 27 x 2 − 324 v y + 27 y 2 − 81 z + 270 v 2 z − 54 v y z − 18 z 2 +<br />

18 v 2 z 2 − z 3 , −1458 v − 27 v x 2 + 486 y + 324 v 2 y−<br />

27 v y 2 − 810 v z + 135 y z + 54 v 2 y z − 144 v z 2 + 9 y z 2 − 8 v z 3 ,<br />

1458 v 2 − 324 x 2 + 243 v 2 x 2 − 4860 v y + 1134 y 2 + 243 v 2 y 2 + 972 z+<br />

162 v 2 z − 108 x 2 z − 1458 v y z + 135 y 2 z + 540 z 2 − 108 v y z 2 +<br />

84 z 3 + 4 z 4 ,<br />

10


(g7)<br />

(g8)<br />

(g9)<br />

(g10)<br />

(g11)<br />

(g12)<br />

(g13)<br />

− 3 v + 2 v 3 + y − v z,<br />

1458 u − 486 x + 162 v 2 x + 27 v x y + 27 u y 2 + 810 u z − 216 x z + 36 v 2 x z+<br />

144 u z 2 − 24 x z 2 + 8 u z 3 ,<br />

54 v 2 + 9 u x − 9 v y − 9 z + 6 v 2 z − z 2 ,<br />

18 u v − 3 v x − 3 u y + 4 u v z,<br />

81 u − 27 x + 9 v 2 x + 9 u v y + 27 u z − 6 x z + 2 u z 2 ,<br />

9 u + 6 u v 2 − 3 x + u z,<br />

3 u 2 + 3 v 2 − z}<br />

The equation of the smallest variety V containing the Enneper surface is g1(x, y, z) = 0.<br />

(b) Over C, use the Extension Theorem to prove that the above equations (3.3.8.1) parametrize the<br />

entire surface V . Hint: There are lots of polynomials in the Gröbner basis. Keep looking – you will find<br />

what you need.<br />

Solution. Apparently with the order u > v > x > y > z<br />

I = 〈g1, . . . , g13〉<br />

I1 = 〈g1, g2, g3, g4, g5, g6, g7〉<br />

I2 = 〈g1〉,<br />

Now given a point (x, y, z) ∈ V(I2) on the variety V the Extension Theorem states that over C <strong>we</strong> can find a<br />

v such that (v, x, y, z) ∈ V(I1) because the coefficient of the hignest po<strong>we</strong>r of v in g7 is the constant 2 which<br />

never vanishes. Starting with <strong>this</strong> (v, x, y, z) ∈ V(I1) the Extension theorem guarantees that <strong>we</strong> can find a<br />

complex number u such that (u, v, x, y, z) ∈ V(I) because the coefficient of the highest po<strong>we</strong>r of u in g13 is<br />

3 which never vanishes. Thus every (x, y, z) ∈ V gives rise to a (u, v, x, y, z) ∈ V(I) which hence satisfies<br />

(3.3.8.1) and shows that <strong>this</strong> point (x, y, z) is “covered” by the parametrization (3.3.8.1).<br />

<strong>§3.</strong>3.9.<br />

The Whitney umbrella surface is given parametrically by<br />

(3.3.9.1)<br />

x = uv,<br />

y = v,<br />

z = u 2 .<br />

(A picture of <strong>this</strong> surface is given in CLO but <strong>we</strong> leave it out.)<br />

(a) Find the equation of the smallest variety containing the Whitney umbrella.<br />

Solution. Again using Mathematica, a Gröbner basis for the ideal I = 〈x − uv, y − v, z − u 2 〉 computed<br />

with lex order and u > v > x > y > z is<br />

{x 2 − y 2 z, v − y, −x + u y, u x − y z, u 2 − z}.<br />

So the equation of the smallest variety containing the Whitney umbrella is x 2 − y 2 z = 0 .<br />

11


(b) Show that the parametrization fills up the variety over C but not over R. Over R, exactly what<br />

points are omitted?<br />

Solution. Here <strong>we</strong> have<br />

I = 〈x 2 − y 2 z, v − y, −x + u y, u x − y z, u 2 − z〉,<br />

I1 = 〈x 2 − y 2 z, v − y〉,<br />

I2 = 〈x 2 − y 2 z〉.<br />

A point (x, y, z) ∈ V = V(I2) can be augmented to a point (v, x, y, z) ∈ V(I1) because the coefficient of the<br />

highest po<strong>we</strong>r of v in the polynomial v − y is 1 which never vanishes. Note here that if x, y, z are real, then<br />

so is v.<br />

Continuing, a point (v, x, y, z) ∈ V(I1) can be augmented to a point (u, v, x, y, z) ∈ V(I) because the<br />

coefficient of the highest po<strong>we</strong>r of u in u 2 − z is 1 which (again!) never vanishes. Here if z < 0 the<br />

augmentation requires that u be non real. So it is the points (x, y, z) with x 2 − y 2 z = 0 and z < 0 which are<br />

not reached by the real form of the parametrization (3.3.9.1).<br />

(c) Show that the parameters u and v are not always uniquely determined by x, y, z. Find the points<br />

where uniqueness fails and explain how your ans<strong>we</strong>r relates to the picture.<br />

Solution. Given (x, y, z) ∈ V(I2) = V , v is determined by the equation v − y = 0. u is determined by the<br />

equations −x + u y = 0, u x − y z = 0, and u 2 − z = 0. u too is certainly unique unless both y = 0 and<br />

x = 0, that is u will be unique except possibly for points on the z-axis. There, in fact, ( √ z, 0, 0, 0, z) and<br />

(− √ z, 0, 0, 0, z) are points extending (0, 0, z) with two different values for the coordinate u. I guess in the<br />

picture shown that the z-axis is the set of singular points where the surface self-intersects.<br />

<strong>§3.</strong>3.10.<br />

Consider the curve in C n parametrized by xi = fi(t), where f1, . . . , fn are polynomials in C[t]. This<br />

gives the ideal<br />

I = 〈x1 − f1(t), . . . , xn − fn(t)〉 which is an ideal in C[t, x1, . . . , xn].<br />

(a) Prove that the parametric equations fill up all of the variety V(I1) ⊂ C n .<br />

Restatement. What <strong>we</strong> are to show is that if (a1, . . . , an) is a zero of each h(x1, . . . , xn) of the form<br />

h(x1, . . . , xn) = w1(t, x1, . . . , xn)(x1 − f1(t)) + · · · + wn(t, x1, . . . , xx)(xn − fn(t)),<br />

where wi ∈ C[t, x1, . . . , xn], 1 ≤ i ≤ n, the main requirement being that h doesn’t involve the variable t,<br />

then there is a u ∈ C, with a1 = f1(u), . . . , an = fn(u).<br />

As an Illustration. Consider the curve x = t, y = t 2 . The point (2, 4) is a zero of each<br />

One of these h’s is, for example,<br />

h(x, y) = b(t, x, y)(x − t) + c(t, x, y)(y − t 2 ), b, c ∈ C[t, x, y].<br />

y − x 2 = −(x + t)(x − t) + 1 · (y − t 2 ).<br />

The result <strong>we</strong> are to establish says that there is a u with (2, 4) = (u, u 2 ). True here with u = 2 of course.<br />

Solution. We are going to cheat and use the Extension Theorem. Here with the variables ordered by<br />

t > x1 > x2 > · · · > xn <strong>we</strong> are asked to extend the partial solution (a1, . . . , an) from I1 to I, from (a1, . . . , an)<br />

to (u, a1, . . . , an), and the Extension Theorem says that <strong>this</strong> is possible (over C) if the coefficients of the<br />

highest po<strong>we</strong>rs of t in the polynomials xi − fi(t) do not have a common zero. <strong>In</strong> <strong>this</strong> case, where t is just a<br />

single variable, these coefficients are constants (they do not involve x1, . . . , xn); so either (i) one of them is<br />

nonzero in which case the Extension Theorem guarantees the existence of a u or (ii) all of them are zero in<br />

which case the statement to be shown is trivial.<br />

12


(b) Show that the conclusion of part (a) may fail if <strong>we</strong> let f1, . . . , fn be rational functions. Hint: See §3<br />

of <strong>Chapter</strong> 1.<br />

Solution. Consider the parametric equations<br />

(3.3.10.b1)<br />

x = t<br />

1 + t ,<br />

y = 1 − 1<br />

.<br />

t2 A simple calculation verifies that t = x<br />

1−x and substituting <strong>this</strong> last expression for t in the second equation<br />

yields y = 1 − <br />

1−x 2 2<br />

x or x y − 2x + 1 = 0.<br />

Remark. The Mathematica command sequence<br />

H={(1+t)*x-t,t^2y-t^2+1}<br />

GroebnerBasis[H,{t,x,y}]<br />

gives the result<br />

{1 − 2 x + x 2 y, 1 − t + t y − x y, −t + x + t x},<br />

which is in keeping with the preceding calculations to eliminate t from (3.3.10.b1).<br />

Anyway, the point (1, 1) is on the curve x 2 y−2x+1 = 0 but cannot be represented in the form (3.3.10.b1)<br />

by any t ∈ C.<br />

(c) Even if all of the fj’s are polynomials, show that it may not be true that the parametric equations<br />

fill up all of the variety V(I1) if <strong>we</strong> work over R.<br />

Solution. Let the parametric equations be: x = t 2 , y = t 2 . Then I = 〈x − t 2 , y − t 2 〉 and I1 = 〈x − y〉. The<br />

point (−1, −1) ∈ V(I1), but can not be represented in the form (t 2 , t 2 ) for any real t.<br />

<strong>§3.</strong>3.11.<br />

This problem is concerned with the proof of<br />

Theorem 3.3.4 (Theorem 2) (Rational <strong>Implicitization</strong>). If k is an infinite field, consider<br />

the rational parametrization F : (k m − W ) → k n given by<br />

(t1, . . . , tm) F<br />

<br />

f1(t1, . . . , tm)<br />

↦→<br />

g1(t1, . . . , tm) , . . . , fn(t1,<br />

<br />

. . . , tm)<br />

,<br />

gn(t1, . . . , tm)<br />

where fi, gi ∈ k[t1, . . . , tm], and W = V(g1(t) · · · gn(t)). Let J be the ideal<br />

J = 〈g1(t)x1 − f1(t), . . . , gn(t)xn − fn(t), 1 − yg1(t) · · · gn(t)〉 ⊂ k[y, t1, . . . , tm, x1, . . . , xn],<br />

and let Jm+1 = J ∩ k[x1, . . . , xn] be the (m + 1)-st elimination ideal. Then V(Jm+1) is the smallest<br />

variety in k n containing F (k m − W ).<br />

Restatement of Conclusion. If f ∈ k[x1, . . . , xn] vanishes on F (k m − W ) then f ∈ Jm+1.<br />

(a) Let k be an infinite field and let f, g ∈ k[t1, . . . , tm]. Assume that g = 0 and that f vanishes on<br />

k m − V(g). Prove that f is the zero polynomial. Hint: Consider the product fg.<br />

Solution. The product fg is zero, both on V(g) and k m − V(g); so it is zero on k m . Since k is an infinite<br />

field it follows that fg = 0 is the zero polynomial. But k[t1, . . . , tm] is an integral domain; so either (i) g = 0<br />

or (ii) f = 0. Since (i) has been ruled out it must be that f = 0 is the zero polynomial.<br />

(b) Prove Theorem 2 using the hints given in the text.<br />

13


Solution. We begin by restating the Closure Theorem<br />

Theorem 3.2.0.3. (Theorem 3) (The Closure Theorem). Let V = V(f1, . . . , fs) ⊂ C n and<br />

let Iℓ be the ℓ-th elimination ideal of 〈f1, . . . , fs〉. That is, Iℓ = 〈f1, . . . , fs〉 ∩ C[xℓ+1, . . . , xn]. Then:<br />

(i) V(Iℓ) is the smallest affine variety containing πℓ(V ) ⊂ Cn−ℓ .<br />

(ii) When V = ∅, there is an affine variety W ⊂<br />

= V(Iℓ) such that V(Iℓ) − W ⊂ πℓ(V ).<br />

(iii) There are affine varieties Zi ⊂ Wi ⊂ Cn−ℓ with 1 ≤ i ≤ m such that<br />

m<br />

πℓ(V ) = (Wi − Zi).<br />

i=1<br />

Note. ALL OF THE CLOSURE THEOREM THAT WE USE IN THIS PROOF is Part (i) which<br />

states: “If a polynomial f ∈ C[xℓ+1, . . . , xn] vanishes on πℓ(V ) ⊂ C n−ℓ , then f ∈ I∩C[xℓ+1, . . . , xn].”<br />

Proof of Rational <strong>Implicitization</strong> Theorem. First (by way of review) <strong>we</strong> let<br />

g(t) = g1(t)g2(t) · · · gn(t) be the product of the “denominator polynomials” g1, . . . , gn and set W = V(g).<br />

Then the maps<br />

j : k m − W → k 1+m+n and F : k m − W → k n are defined by<br />

(3.3.11.1)<br />

(3.3.11.2)<br />

(t1, t2, . . . , tm) j<br />

<br />

1 f1(t) fn(t)<br />

↦→ , t, , . . . , = (y, s, x), t ∈ k<br />

g(t) g1(t) gn(t)<br />

m − W.<br />

(t1, t2, . . . , tm) F<br />

<br />

f1(t) fn(t)<br />

↦→ , . . . , , t ∈ k<br />

g1(t) gn(t)<br />

m − W.<br />

We will use y, s, x for the coordinate variables on k 1+m+n , where s = (s1, . . . , sm) and x = (x1, . . . , xn). Let<br />

J = 〈1 − yg(s), g1(s)x1 − f1(s), . . . , gn(s)xn − fn(s)〉. J is an ideal in the ring k[y, s, x]. Our first contention<br />

is:<br />

Contention 1. j(k m − W ) = V(J).<br />

Proof of Contention 1: (⊂): If q ∈ j(km − W ), then q has the form on the right of (3.3.11.1) for some<br />

t. We evaluate the polynomials in the basis for J at q, using the facts that s(q) = t, gi(s)(q) = gi(t), and<br />

fi(s)(q) = fi(t). This gives<br />

1<br />

(1 − yg(s) (q) = 1 − · g(t) = 0,<br />

g(t)<br />

(gi(s)xi − fi(s))(q) = gi(t) · fi(t)<br />

gi(t) − fi(t) = 0.<br />

So it follows that j(k m − W ) ⊂ V(J).<br />

Thus<br />

(⊃): Suppose now that p = (b, r, z) ∈ V(J). Then<br />

0 = (1 − yg(s))(p) = 1 − bg(r) =⇒ g(r) = 0 so r ∈ k m − W and b = 1<br />

g(r)<br />

and for each 1 ≤ i ≤ n, gi(r) = 0 and<br />

0 = (gi(s)xi − fi(s))(p) = gi(r)zi − fi(r) =⇒ zi = fi(r)<br />

gi(r) .<br />

(b, r, z) =<br />

This completes the proof of Contention 1.<br />

<br />

1 f1(r) fn(r)<br />

, r, , . . . , ∈ j(k<br />

g(r) g1(r) gn(r)<br />

m − W ).<br />

14


The case where k is algebraically closed: A direct application of (i) of the Closure Theorem with<br />

ℓ = m + 1 then shows that V(Jm+1) is the smallest affine variety which contains πm+1 (V(J)). Now notice<br />

that<br />

<br />

f1(r) fn(r)<br />

πm+1 (V(J)) = , . . . , : r ∈ k<br />

g1(r) gn(r)<br />

m <br />

− W = F (k m − W ).<br />

With <strong>this</strong> identification the proof of the Rational <strong>Implicitization</strong> Theorem when k is algebraically closed is<br />

complete.<br />

Next suppose that k ⊂ K where K is algebraically closed. Since k may be strictly smaller than K <strong>we</strong><br />

cannot use the Closure Theorem directly; so our strategy will be to switch back and forth bet<strong>we</strong>en k and K<br />

and <strong>we</strong> will use the subscript k or K (in parentheses) to keep track of which field <strong>we</strong> are working with. Thus<br />

V (k)(Jm+1) is the variety in k n where the polynomials of Jm+1 vanish and V (K) (Jm+1) is the larger set of<br />

solutions in K n . Let<br />

H = {1 − yg(s), g1(s)x1 − f1(s), . . . , gn(s)xn − fn(s)},<br />

then<br />

V (k)(Jm+1) = x ∈ k n : for some (y, s) ∈ k × k m , the polynomials H vanish on (y, s, x) ;<br />

V (K) (Jm+1) = x ∈ K n : for some (y, s) ∈ K × K m , the polynomials H vanish on (y, s, x) .<br />

Since the polynomials H which generate J don’t themselves change V (k)(Jm+1) ⊂ V (K) (Jm+1). (Note. A<br />

way to get a basis for Jm+1 or Jm+1 is to start with the set of polynomials H and using these polynomials,<br />

all of whose coefficients are in k (by the way), to compute a Gröbner basis G for the ideal J that they<br />

generate in k[y, s, x]. G is unaffected by whether the computaions are carried out using k as the field or<br />

using K because computing a Gröbner basis will never take us out of the field of coefficients of the starting<br />

ideal basis. Then a basis for Jm+1 consists of G ∩ k[x1, . . . , xn] and nowhere does it make any difference<br />

whether the computations are carried out over k or over K. So G ∩ k[x] = G ∩ K[x].)<br />

This discussion shows that the same set of polynomials in k[x] serve as a Gröbner basis for the ideal<br />

Jm+1 in k[x] and for the ideal Jm+1 in K[x]. Let these polynomials be<br />

Collecting <strong>this</strong> information,<br />

{b1(x), . . . , bs(x)} = G ∩ k[x] = G ∩ K[x].<br />

(Jm+1) Jm+1 = { ˜ ψ ∈ K[x]: ˜ ψ(x) = ˜ h1(x)b1(x) + · · · + ˜ hs(x)bs(x), ˜ hi(x) ∈ K[x], 1 ≤ i ≤ s}<br />

and<br />

(Jm+1) Jm+1 = {ψ ∈ k[x]: ψ(x) = h1(x)b1(x) + · · · + hs(x)bs(x), hi(x) ∈ k[x], 1 ≤ i ≤ s}.<br />

Using the fact that these hi’s and ˜ hj’s can be chosen rather arbitrarily, <strong>we</strong> have established that<br />

and<br />

a point q ∈ K n is in V (K) (Jm+1) iff bi(q) = 0, 1 ≤ i ≤ s<br />

a point p ∈ k n is in V (k)(Jm+1) iff bi(p) = 0, 1 ≤ i ≤ s.<br />

We need to prove that V (k)(Jm+1) is the smallest variety in k n containing F (k m − W (k)). Since K is<br />

algebraically closed <strong>we</strong> can pick up here applying the closure theorem to state that: “V (K) (Jm+1) has been<br />

shown to be the smallest variety in K n containing F (K m − W K )”.<br />

I don’t think <strong>we</strong> actually need to do <strong>this</strong>, but to make sure our argument is complete <strong>we</strong> next show that<br />

the variety V (k)(Jm+1) actually contains F (k m −W (k)) and that V (K) (Jm+1) actually contains F (K m −W (K) ).<br />

15


Contention 2. (i) F (k m − W (k)) = πm+1(V (k)) ⊂ V (k)(Jm+1) and<br />

(ii) F (K m − W (K) ) = πm+1(V (K) ) ⊂ V (K) (Jm+1).<br />

Proof of Contention 2. Suppose f ∈ Jm+1 and p ′ = πm+1(p) with p ∈ V (k). The polynomial f only<br />

involves the variables x1, . . . , xn; so <strong>we</strong> can regard it as being a polynomial f in k[x1, . . . , xn] or as being<br />

a polynomial ˜ f ∈ k[y, s1, . . . , sm, x1, . . . , xn] which doesn’t involve y or s. <strong>In</strong> the latter case <strong>we</strong> have the<br />

relations ˜ f = f ◦ πm+1 and ˜ f ∈ Jm+1. Together with p ∈ V (k)(J) and ˜ f ∈ J <strong>we</strong> have 0 = ˜ f(p) =<br />

(f ◦ πm+1)(p) = f (πm+1(p)) = f(p ′ ), which proves the contention.<br />

Let f ∈ k[x1, . . . , xn] be any polynomial vanishing on F (k m − W (k)). To complete the proof <strong>we</strong> must<br />

show that f ∈ Jm+1 or, equivalently, that f vanishes on V (k)(Jm+1). First, note that f ◦ F vanishes on all<br />

of km − W (k). Since f is a polynomial in the variables x1, . . . , xn and<br />

<br />

f1<br />

F =<br />

, . . . ,<br />

g1<br />

fn<br />

gn<br />

are rational functions in t1, . . . , tm. It follows that f ◦ F is a rational function in the variables t1, . . . , tm.<br />

Now<br />

Contention 3.The only rational function in t1, . . . , tm which vanishes on all of k m −W (k) is the zero rational<br />

function.<br />

Proof of contention 3. Suppose <strong>this</strong> rational function has the form p(t)<br />

q(t) . The polynomial t ↦→ g(t)p(t)<br />

vanishes on km <br />

− W (k) ∪ W(k) = km . Since k is an infinite field <strong>this</strong> implies that g(t)p(t) = 0 ∈ k[t]. Since<br />

g(t) is not the zero polynomial and k[t] is an integral domain, <strong>this</strong> implies that p(t) is the zero polynomial<br />

is the zero rational function.<br />

in k[t] and p(t)<br />

q(t)<br />

We need one more fact to carry the argument on smoothly.<br />

Contention 4. k n − W (k) ⊂ K n − W (K) .<br />

Proof of contention 4.<br />

k n − W (k) = k n ∩ {ζ ∈ k n : g(ζ) = 0}<br />

⊂ K n ∩ {ζ ∈ K n : g(ζ) = 0}<br />

= K n − W (K) .<br />

Now to the proof proper. When f ∈ k[x1, . . . , xn] each line below implies the line which follows it.<br />

f vanishes on F (k m − W (k));<br />

f ◦ F vanishes on k m ;<br />

f ◦ F is the zero rational function ;<br />

f ◦ F vanishes on K m ;<br />

f vanishes on F (K m − W (K) ); <strong>this</strong> implies the next line by Statement<br />

f vanishes on {q ∈ K n − W (K) : Jm+1(q) = {0}};<br />

(i) of the Closure Theorem;<br />

f vanishes on {q ∈ K n − W (K) : b1(q) = 0, . . . , bs(q) = 0}; <strong>this</strong> implies the next line<br />

f vanishes on {p ∈ k n − W (k) : b1(p) = 0, . . . , bs(p) = 0};<br />

f vanishes on {p ∈ k n − W (k) : Jm+1(p) = {0}};<br />

V (k)(Jm+1) ⊂ V (k)(f). where the varieties are in k n .<br />

because of contention 4<br />

which proves that V (k)(Jm+1) is the smallest variety of k n containing F (k m −W (k)), and completes the proof<br />

of the theorem.<br />

16


<strong>§3.</strong>3.12.<br />

Consider the following rational parametrization:<br />

(3.3.3) (u, v) F<br />

<br />

2 u v2<br />

↦→ , , u .<br />

v u<br />

For simplicity, let k = C . Also let I = 〈vx − u 2 , uy − v 2 , z − u〉 be the ideal obtained by “clearing<br />

denominators”.<br />

(a) Show that I2 = 〈zx 2 y − z 4 〉<br />

Solution. As <strong>we</strong> found before the Mathematica commands<br />

h={v*x-u^2,u*y-v^2,z-u}<br />

GroebnerBasis[h,{u,v,x,y,z}]<br />

executed in sequence yield<br />

{x 2 y z − z 4 , − (x y z) + v z 2 , v x − z 2 , v 2 − y z, u − z},<br />

and <strong>we</strong> can read off I2 = 〈x 2 y z − z 4 〉 from <strong>this</strong>.<br />

(b) Show that the smallest variety in C 5 containing i(C 2 − W ) is<br />

where i: (C 2 − W ) −→ C 5 is the map<br />

V(vx − u 2 , uy − v 2 , z − u, x 2 y − z 3 , vz − xy),<br />

(u, v)<br />

<br />

i<br />

−→<br />

u, v, u2<br />

v<br />

<br />

v2<br />

, , u .<br />

u<br />

Hint: Show that i(C 2 − W ) = π1(V(J)), and then use the Closure Theorem.<br />

Solution. Following the hint, let j : C 2 −→ C 6 be the map<br />

(u, v)<br />

j<br />

−→<br />

1<br />

uv<br />

, u, v, u2<br />

v<br />

<br />

v2<br />

, , u .<br />

u<br />

and J = 〈1 − yuv, vx − u 2 , uy − v 2 , z − u〉. It is clear that i = π1 ◦ j and <strong>we</strong> know from previous<br />

discussion ( Viz.,the proof of the Rational <strong>Implicitization</strong> Theorem given in the solution to Exercise 3.3.11)<br />

that V(J) = j(C 2 − W ). Applying π1 to <strong>this</strong> last equation gives π1 (V(J)) = (π1 ◦ j)(C 2 − W ) = i(C 2 − W ).<br />

Now the Closure Theorem guarantees that V(J1) is the smallest affine variety containing π1 (V(J)). So V(J1)<br />

is the smallest variety containing i(C 2 − W ). This leaves us the problem of identifying V(J1). This task is<br />

made easier by Mathematica. The Mathematica command sequence:<br />

H={1 − t u v, v x − u 2 , u y − v 2 , z − u}<br />

GroebnerBasis[H,{t,u,v,x,y,z}]<br />

produces the output<br />

So<br />

{x 2 y − z 3 , − (x y) + v z, v x − z 2 , v 2 − y z, u − z, −x + t z 3 , −v + t y z 2 , −1 + t x y}<br />

J = 〈x 2 y − z 3 , − (x y) + v z, v x − z 2 , v 2 − y z, u − z, −x + t z 3 , −v + t y z 2 , −1 + t x y〉<br />

J1 = 〈x 2 y − z 3 , − (x y) + v z, v x − z 2 , v 2 − y z, u − z〉<br />

We can rewrite J1, rearranging terms and making free use of the relation u − z = 0, as<br />

so<br />

J1 = 〈v x − z 2 , v 2 − y u, u − z, x 2 y − z 3 , − (x y) + v z〉;<br />

V(J1) = V(v x − z 2 , v 2 − y u, u − z, x 2 y − z 3 , − (x y) + v z)<br />

is the smallest variety in C 5 containing i(C 2 − W ).<br />

17


Remark. Theorem 3.2.0.2, Theorem 2 of <strong>§3.</strong>2, tells us that<br />

V(J1) = π1 (V(J)) ∪ (V(g1, . . . , gs) ∩ V(J1)) ,<br />

where in <strong>this</strong> case the gi’s are the coefficients of the highest po<strong>we</strong>r of the variable t in a polynomial basis<br />

for J. Disregarding those polynomials which don’t involve t, and whose “coefficients of highest po<strong>we</strong>r of<br />

t” don’t add anything anyway because they form a basis for J1 and their zeros appear in π1(V(J)), these<br />

polynomials are z 3 , yz 2 and xy. Considering <strong>this</strong> it is not clear to me at the outset that V(J1) is actually<br />

equal to π1(V(J)). No matter, it is still the smallest variety containing π1(V(J)) and that is all part (b) of<br />

Exercise 3.3.12 actually claims.<br />

(c) Show that {(0, 0, x, y, 0): x, y arbitrary} ⊂ V(I) and conclude that V(I) is not the smallest variety<br />

containing i(C 2 − W ).<br />

Solution. Remembering that I = 〈vx − u 2 , uy − v 2 , z − u〉 it is easy to check that<br />

{(0, 0, x, y, 0): x, y arbitrary} ⊂ V(I). On the otherhand Since<br />

J1 = 〈v x−z 2 , v 2 −y u, u−z, x 2 y −z 3 , − (x y)+v z〉, the only points of the set {(0, 0, x, y, 0): x, y arbitrary}<br />

which are in V(J1) are those for which xy = 0. <strong>In</strong> particular, (0, 0, 1, 1, 0) ∈ V(I) − V(J1); so V(I) = V(J1)<br />

and by part(b) of <strong>this</strong> exercise V(I) can’t be V(J1), the smallest variety containing i(C 2 − W ).<br />

(d) Determine exactly which portion of x 2 y − z 3 = 0 is parametrized by (3.3.3).<br />

Suppose (x, y, z) satisfies x 2 y = z 3 .<br />

Case z = 0. Then y = 0, yz > 0 and there are two values for x, namely, x = ±<br />

(3.3.12.d1)<br />

(x, y, z): x 2 y = z 3 , z = 0 =<br />

<br />

±<br />

<br />

<br />

z3 y<br />

z3 <br />

, y, z<br />

y<br />

: yz > 0 .<br />

. Thus the points<br />

<br />

2<br />

u v2<br />

Each of these points has the form v , u , u . To see <strong>this</strong>, given a point with the form (3.3.12.d1) set u = z<br />

and v = ± √ yz. With these choices for u and v,<br />

u 2<br />

v<br />

<br />

v2<br />

z<br />

, , u = ±<br />

u 3<br />

<br />

, y, z .<br />

y<br />

So all the points of (3.3.12.d1) are “covered” by the parametrization.<br />

Case z = 0. Here x2y = z3 is the graph of<br />

the x and y coordinate axes on the coordinate plant z = 0. None<br />

2<br />

u<br />

of these points have the form<br />

<strong>§3.</strong>3.13.<br />

v<br />

v2 , u , u<br />

Given a rational parametrization t ↦→<br />

; so they are not covered by the parametrization.<br />

f1(t)<br />

g1(t)<br />

<br />

fn(t)<br />

, . . . , gn(t) , there is one case where the naive ideal I =<br />

〈g1(t)x1 − f1(t), . . . , gn(t)xn − fn(t)〉 ⊂ k[t, x] obtained by “clearing denominators” gives the right ans<strong>we</strong>r.<br />

Suppose that t = (t1) = (t), i.e. there is only one value of t. We can assume for each i that fi(t) and gi(t)<br />

are relatively prime in k[t]; so, in particular, they have no common roots. If I ⊂ k[t, x1, . . . , xn] is as above,<br />

then prove that V(I1) is the smallest variety containing F (k − W ), where as usual g = g1 · · · gn ∈ k[t] and<br />

W = V(g) ⊂ k. Hint: Show that i(k − W ) = V(I), where i: (k − W ) → k n+1 is given by<br />

t i<br />

<br />

↦→<br />

t, f1(t) fn(t)<br />

, . . . ,<br />

g1(t) gn(t)<br />

and adapt the proof of the Polynomial <strong>Implicitization</strong> Theorem.<br />

Solution. We prove first the following:<br />

18<br />

<br />

,


Contention 1. i(k − W ) = V(I).<br />

Proof of contention 1. (⊃): Suppose p = (t, x1, . . . , xn) ∈ V(I). Then for each i, 1 ≤ i ≤ n, gi(t)xi−fi(t) =<br />

0. Since gi and fi have no common zeros it must be that gi(t) = 0, 1 ≤ i ≤ n; so <strong>we</strong> have at the start that<br />

t ∈ k − W . But then solving the relations gi(t)xi − fi(t) = 0 for xi shows, since t ∈ k − W , that<br />

<br />

p = (t, x1, . . . , xn) = t, f1(t)<br />

<br />

fn(t)<br />

, . . . , ∈ i(k − W ).<br />

g1(t) gn(t)<br />

<br />

(⊂): If t ∈ k − W , then i(t) =<br />

t, f1(t)<br />

g1(t)<br />

(s, x) ↦→ gi(s)xi − fi(s) vanish on i(t). This shows that i(k − W ) ⊂ V(I).<br />

<br />

fn(t)<br />

, . . . , gn(t) and it is an easy matter to check that the polynomials<br />

Contention 2. If k is algebraically closed, then V(I1) is the smallest variety containing F (k − W ), where<br />

F = π1 ◦ i: (k − W ) → k n is the map<br />

t F<br />

<br />

f1(t) fn(t)<br />

↦→ , . . . , .<br />

g1(t) gn(t)<br />

Proof of contention 2. We know from the Closure Theorem that V(I1) is the smallest variety containing<br />

π1(V(I)). Now π1 (V(I)) = π1 (i(k − W )) = (π1 ◦ i) (k − W ) = F (k − W ).<br />

Next suppose that k ⊂ K where K is algebraically closed. Since k may be strictly smaller than K <strong>we</strong><br />

cannot use the Closure Theorem directly; so our strategy will be to switch back and forth bet<strong>we</strong>en k and K<br />

and <strong>we</strong> will use the subscript k or K (in parentheses) to keep track of which field <strong>we</strong> are working with. Thus<br />

V (k)(I1) is the variety in k n where the polynomials of I1 vanish and V (K) (I1) is the larger set of solutions in<br />

K n . (Note. A way to get a basis for I1 is to start with the set of polynomials<br />

(3.3.13.1) {gi(s)xi − fi(s): 1 ≤ i ≤ n}<br />

and using these polynomials, whose coefficients are in k, to compute a Gröbner basis G for the ideal I that<br />

they generate in k[s, x]. G is unaffected by whether the computations are carried out using k as the field or<br />

using K because computing a Gröbner basis will never take us out of the field of coefficients of the starting<br />

ideal basis. Then a basis for I1 consists of G∩k[x1, . . . , xn] and nowhere does it make any difference whether<br />

the computations are carried out over k or over K.)<br />

We need to prove that V (k)(I1) is the smallest variety in k n containing F (k − W (k)). We know from<br />

contention 2 that V (K) (I1) is the smallest variety in K n containing F (K − W (K) ).<br />

Put in other words: Let Z (k) = V (k)(h1, . . . , hs) ⊂ kn be any variety of kn containing F (k − W (k));<br />

so F (k − W (k)) ⊂ Z (k). We must show that V (k)(I1) ⊂ Z (k). First, note that hi = 0 on Z (k) and, hence,<br />

hi = 0 on the smaller set F (k − W (k)). This shows that each hi ◦ F vanishes on all of k − W (k). But hi is a<br />

polynomial in the variables x1, . . . , xn and<br />

<br />

f1<br />

F = , . . . ,<br />

g1<br />

fn<br />

<br />

gn<br />

are rational functions in t. It follows that hi ◦F ∈ k(t), the field of rational functions in t, and hi ◦F vanishes<br />

on k − W (k). To continue <strong>this</strong> arguement <strong>we</strong> make use of contention 3 below in the special case where m = 1.<br />

We can do <strong>this</strong> because k is infinite and W (k) is finite; so k − W (k) is still infinite.<br />

Contention 3.Any rational function in t1, . . . , tm which vanishes on all of k m − W (k) is the zero rational<br />

function.<br />

Proof of contention 3. Suppose <strong>this</strong> rational function has the form p(t)<br />

q(t) . The polynomial t ↦→ g(t)p(t)<br />

vanishes on km <br />

− W (k) ∪ W(k) = km . Since k is an infinite field <strong>this</strong> implies that g(t)p(t) = 0 ∈ k[t]. Since<br />

g(t) is not the zero polynomial and k[t] is an integral domain, <strong>this</strong> implies that p(t) is the zero polynomial<br />

is the zero rational function.<br />

in k[t] and p(t)<br />

q(t)<br />

19


Picking up the proof where <strong>we</strong> left off, contention 3 guarantees that hi ◦ F is the zero rational function;<br />

so hi ◦ F vanishes not only on k − W (k) but on K − W (K) , where K ⊃ k is algebraically closed. This means<br />

that the hi’s vanish on F (K −W (K) ). Contention 2 then guarantees that V (K) (I1) ⊂ V (K) (h1, . . . , hs) because<br />

V (K) (I1) is the smallest variety in K n which contains F (K − W (K) ). It follows that<br />

V (k)(I1) = k n ∩ V (K) (I1) ⊂ K n ∩ V (K) (h1, . . . , hs) = V (k)(h1, . . . , hs).<br />

This completes the proof of the fact that when there is just one parameter t, V(I1) is the smallest variety in<br />

k n which contains F (k − W ).<br />

<strong>§3.</strong>3.14.<br />

The folium of Descartes can be parametrized by<br />

x = 3t<br />

,<br />

1 + t3 y = 3t2<br />

.<br />

1 + t3 (a) Find the equation of the folium. Hint: Use Exercise 3.3.13.<br />

Solution. Using Mathematica a Gröbner basis for I = 〈(1 + t 3 )x − 3t, (1 + t 3 )y − 3t 2 〉 is<br />

{x 3 − 3 x y + y 3 , x 2 − 3 y + t y 2 , t x − y, −3 t + x + t 2 y}.<br />

So the equation of the folium is x 3 − 3 x y + y 3 = 0 .<br />

(b) Over C or R, show that the above parametrization covers the entire curve.<br />

Solution. If (x, y) ∈ V(I1), i.e. satisfies x 3 − 3 x y + y 3 = 0, let us try to extend to a point (t, x, y) ∈ V(I).<br />

The Extension Theorem guarantees that the extension will work over C unless (x, y)V(y 2 , x, y) = V(x, y).<br />

That is except possibly at the point (0, 0), but if x = 0, y = 0 the extension is easy. It is (t, x, y) = (0, 0, 0).<br />

For the real case <strong>we</strong> must reason differently and our reasoning here covers both the real and complex<br />

cases. Let us try to extend (x, y) ∈ V(I1), and thereby satisfying x3 − 3xy + y3 = 0, to a (t, x, y) ∈ V(I)<br />

satisfying the four equations associated with the polynomial basis of I listed above in the first offset line.<br />

If x = 0, then y = 0 and an/the appropriate extension is to set t = 0 giving (0, 0, 0) ∈ V(I).<br />

If x = 0, put t = y<br />

x . y<br />

x , x, y is easily seen to be a zero of the polynomials in the offset line.<br />

<strong>§3.</strong>3.15.<br />

<strong>In</strong> Exercise 16 to §3 of <strong>Chapter</strong> 1, <strong>we</strong> studied the parametric equations over R<br />

x = (1 − t)2x1 + 2t(1 − t)wx2 + t2x3 (1 − t) 2 + 2t(1 − t)w + t2 ,<br />

y = (1 − t)2y1 + 2t(1 − t)wy2 + t2y3 (1 − t) 2 + 2t(1 − t)w + t2 ,<br />

where w, x1, y1, x2, y2, x3, y3 are constants and w > 0. By eliminating t show that these equations describe<br />

a portion of a conic <strong>section</strong>. Recall that a conic <strong>section</strong> is described by an equation of the form<br />

ax 2 + bxy + cy 2 + dx + ey + f = 0.<br />

Hint: <strong>In</strong> most computer algebra systems, the Gröbner basis command allows polynomials to have coefficients<br />

involving symbolic constants like w, x1, y1, x2, y2, x3, y3. Since <strong>we</strong> are over R and the denominators never<br />

vanish, you can use Exercise 3.3.13.<br />

20


Solution. If one runs the Mathematica command sequence<br />

A=(1-t)^2*x1+2*t*(1-t)*w*x2+t^2*x3<br />

B=(1-t)^2*y1+2*t*(1-t)*w*y2+t^2*y3<br />

F=(1-t)^2+2*t*(1-t)*w+t^2<br />

H={F*x-A,F*y-B}<br />

GroebnerBasis[H,{t,x,y},CoefficientDomain->RationalFunctions]<br />

21


The output is (hold your hat!)<br />

{ x1 2 − 4 w 2 x1 x2 + 4 w 2 x2 2 − 2 x1 x3 + 4 w 2 x1 x3 − 4 w 2 x2 x3 + x3 2 y 2 +<br />

x3 2 y1 2 − 4 w 2 x2 x3 y1 y2 + 4 w 2 x1 x3 y2 2 + 4 w 2 x2 2 y1 y3 − 2 x1 x3 y1 y3 − 4 w 2 x1 x2 y2 y3 + x1 2 y3 2 +<br />

x y −2 x1 y1 + 4 w 2 x2 y1 + 2 x3 y1 − 4 w 2 x3 y1 + 4 w 2 x1 y2 − 8 w 2 x2 y2 + 4 w 2 x3 y2 + 2 x1 y3−<br />

<br />

+<br />

4 w 2 x1 y3 + 4 w 2 x2 y3 − 2 x3 y3<br />

y −4 w 2 x2 2 y1 + 2 x1 x3 y1 + 4 w 2 x2 x3 y1 − 2 x3 2 y1 + 4 w 2 x1 x2 y2 − 8 w 2 x1 x3 y2+<br />

<br />

+<br />

4 w 2 x2 x3 y2 − 2 x1 2 y3 + 4 w 2 x1 x2 y3 − 4 w 2 x2 2 y3 + 2 x1 x3 y3<br />

x 2 y1 2 − 4 w 2 y1 y2 + 4 w 2 y2 2 − 2 y1 y3 + 4 w 2 y1 y3 − 4 w 2 y2 y3 + y3 2 +<br />

x −2 x3 y1 2 + 4 w 2 x2 y1 y2 + 4 w 2 x3 y1 y2 − 4 w 2 x1 y2 2 − 4 w 2 x3 y2 2 +<br />

2 x1 y1 y3 − 8 w 2 x2 y1 y3 + 2 x3 y1 y3 + 4 w 2 x1 y2 y3 + 4 w 2 x2 y2 y3 − 2 x1 y3 2 ,<br />

− 2 w x2 y1 2 − x3 y1 2 + 2 w x3 y1 2 + 2 w x1 y1 y2 + 4 w 2 x2 y1 y2 − 2 w x3 y1 y2 − 4 w 2 x1<br />

y2 2 + x1 y1 y3 − 2 w x1 y1 y3 + 2 w x2 y1 y3 − 4 w 2 x2 y1<br />

y3 + x3 y1 y3 + 4 w 2 x1 y2 y3 − x1 y3 2 +<br />

t y −4 w x2 y1 + 4 w 2 x2 y1 + 4 w x3 y1 − 4 w 2 x3 y1 + 4 w x1 y2 − 4 w 2 x1 y2−<br />

<br />

+<br />

4 w x3 y2 + 4 w 2 x3 y2 − 4 w x1 y3 + 4 w 2 x1 y3 + 4 w x2 y3 − 4 w 2 x2 y3<br />

y − (x1 y1) + 2 w x2 y1 + x3 y1 − 2 w x3 y1 − 2 w x1 y2 + 4 w 2 x1 y2 − 4 w 2 x2 y2 + 2 w x3 y2 + x1 y3+<br />

<br />

+<br />

2 w x1 y3 − 4 w 2 x1 y3 − 2 w x2 y3 + 4 w 2 x2 y3 − x3 y3<br />

x y1 2 − 4 w 2 y1 y2 + 4 w 2 y2 2 − 2 y1 y3 + 4 w 2 y1 y3−<br />

4 w 2 y2 y3 + y3 2 + t 2 w x2 y1 2 − 2 w x3 y1 2 −<br />

2 w x1 y1 y2 − 4 w 2 x2 y1 y2 + 2 w x3 y1 y2 + 4 w 2 x3 y1 y2+<br />

4 w 2 x1 y2 2 − 4 w 2 x3 y2 2 + 2 w x1 y1 y3 − 2 w x3 y1 y3−<br />

2 w x1 y2 y3 − 4 w 2 x1 y2 y3 + 4 w 2 x2 y2 y3 + 2 w x3 y2 y3 + 2 w x1 y3 2 − 2 w x2 y3 2 ,<br />

x1 2 − 4 w 2 x1 x2 + 4 w 2 x2 2 − 2 x1 x3 + 4 w 2 x1 x3 − 4 w 2 x2 x3+<br />

x3 2 y + 2 w x1 x2 y1 − 4 w 2 x2 2 y1 + x1 x3 y1 − 2 w x1 x3 y1+<br />

4 w 2 x2 x3 y1 − x3 2 y1 − 2 w x1 2 y2+<br />

4 w 2 x1 x2 y2 + 2 w x1 x3 y2 − 4 w 2 x1 x3 y2 − x1 2 y3+<br />

2 w x1 2 y3 − 2 w x1 x2 y3 + x1 x3 y3 + t x 4 w x2 y1 − 4 w 2 x2 y1−<br />

4 w x3 y1 + 4 w 2 x3 y1 − 4 w x1 y2 + 4 w 2 x1 y2 + 4 w x3 y2 − 4 w 2 x3 y2+<br />

4 w x1 y3 − 4 w 2 x1 y3 − 4 w x2 y3 + 4 w 2 x2 y3<br />

+<br />

x − (x1 y1) − 2 w x2 y1 + 4 w 2 x2 y1 + x3 y1 + 2 w x3 y1 − 4 w 2 x3 y1 + 2 w x1 y2 − 4 w 2 x2 y2−<br />

2 w x3 y2 + 4 w 2 <br />

x3 y2 + x1 y3 − 2 w x1 y3 + 2 w x2 y3 − x3 y3 +<br />

t −2 w x1 x2 y1 + 4 w 2 x2 2 y1 + 2 w x1 x3 y1 − 2 w x2 x3 y1 − 4 w 2 x2 x3 y1 + 2 w x3 2 y1+<br />

2 w x1 2 y2 − 4 w 2 x1 x2 y2 + 4 w 2 x2 x3 y2 − 2 w x3 2 y2 − 2 w x1 2 y3+<br />

2 w x1 x2 y3 + 4 w 2 x1 x2 y3 − 4 w 2 x2 2 y3 − 2 w x1 x3 y3 + 2 w x2 x3 y3<br />

I’ve blocked <strong>this</strong> output into three <strong>section</strong>s each representing a polynomial. Setting the first of these polynomials<br />

equal to zero gives the equation of the variety obtained by eliminating t and the reader can check<br />

without too much difficulty that it is a conic.<br />

22<br />

}

Hooray! Your file is uploaded and ready to be published.

Saved successfully!

Ooh no, something went wrong!